MODERN Flashcards

1
Q

1,”With reference to Indian history, consider the following statements: (2022) 1. The Dutch established their factories/warehouses on the east coast on lands granted to them by Gajapati rulers. 2. Alfonso de Albuquerque captured Goa from the Bijapur Sultanate. 3. The English East India Company established a factory at Madras on a plot of land leased from a representative of the Vijayanagara empire. Which of the statements given above are correct?”,”(a) 1 and 2 only (b) 2 and 3 only (c) 1 and 3 only (d) 1, 2 and 3”,2022

A

1,b,”Statement 1 is incorrect: There is a difference in timeline of the events. The end of Gajapati Dynasty was in 1542 and the United East India Company of the Netherlands, formed in March 1605 by the Charter of Dutch Parliament and founded their first factory in Masulipatnam in Andhra in 1605. Statement 2 is correct: Alfonso de Albuquerque considered as Founder of the Portuguese power in India: captured Goa from Bijapur; persecuted Muslims; captured Bhatkal from Sri Krishna Deva Rai (1510) of Vijayanagara. Statement 3 is correct: The English East India Company established a factory in Madras in 1639 on land leased from representatives of Vijayanagara Empire called the Nayakas.”

How well did you know this?
1
Not at all
2
3
4
5
Perfectly
2
Q

2,”In the first quarter of the seventeenth century, in which of the following was/were the factory/factories of the English East India Company located? (2021) 1. Broach 2. Chicacole 3. Trichinopoly Select the correct answer using the code given below.”,”(a) 1 only (b) 1 and 2 (c) 3 only (d) 2 and 3”,2021

A

2,a,”The English East India Company had a factory at Broach (present-day Bharuch) in Gujarat, India during the seventeenth century. However, Chicacole (present-day Srikakulam) and Trichinopoly (present-day Tiruchirappalli) were not among the locations of the Company’s factories during this period. Establishment of Factories by English East India Company: West Coast: The English India Company established their factories at Agra, Ahmadabad, Baroda and Broach by 1619, under the control of the president and council of the Surat factory which was established in the year 1612. South-eastern Coast: EIC established their factories at Masulipatam in 1611 and Armagaon near Pulicat in 1626. Eastern India: In eastern India EIC established their factories at Hariharpur and Balasore(now Odisha) in 1633, at Hugli in 1651, followed by setting up factories at Patna, Dacca, Kasimbazar in Bengal and Bihar respectively.”

How well did you know this?
1
Not at all
2
3
4
5
Perfectly
3
Q

3,”Which one of the following statements does not apply to the system of Subsidiary Alliance introduced by Lord Wellesley? (2018) (a) To maintain a large standing army at others expense (b) To keep India safe from Napoleonic danger (c) To secure a fixed income for the Company (d) To establish British paramountcy over the Indian States”,2018

A

3,c,”The system aimed to establish British paramountcy over the Indian States and to ensure their military and political subservience to the British East India Company. Securing a fixed income for the company was not the primary objective of the system. Subsidiary Alliance by Wellesley (1798-1805): Allying Indian states maintained the British army at its frontier and paid for their maintenance. Ruler had to post British officials at their court. Ruler couldn’t employ Europeans without the British’s permission. Couldn’t contact enemies to make peace. Maintenance asked was high → Ruler’s couldn’t pay → asked to cede part of their territory.”

How well did you know this?
1
Not at all
2
3
4
5
Perfectly
4
Q

4,”With reference to the history of India, consider the following pairs: (2020) 1. Aurang - In-charge of treasury of the State 2. Banian - Indian agent of the East India Company 3. Mirasidar - Designated revenue payer to the State Which of the pairs given above is/are correctly matched? (a) 1 and 2 only (b) 2 and 3 only (c) 3 only (d) 1, 2 and 3”,2020

A

4,b,”Pair 1 is incorrect: Aurang is a Persian term referring to a warehouse where goods are collected before being sold. Pair 2 is correct: The word Bania (also Vania) is derived from the Sanskrit vanij, meaning ‘a merchant’. The banian acted as an agent for individual East India Company managers and performed a range of functions including, bookkeeping, interpreting, and brokerage, as well as managing his household and personal business activities. Banians served as middlemen for European merchants in Bengal during the British era. Pair 3 is correct: Mirasdar in Urdu means hereditary landowner, co-proprietor and in Persian, miras is inheritance, estate. Under the Ryotwari settlement system in Southern India, the East India Company recognized mirasdars as the only proprietors of land, dismissing tenants’ rights completely. They were prohibited by law from selling their land but it can be leased to tenants for a nominal fee in return for its cultivation. They acted as a designated revenue payer of the state.”

How well did you know this?
1
Not at all
2
3
4
5
Perfectly
5
Q

5,”Which of the following statements correctly explain the impact of the Industrial Revolution on India during the first half of the nineteenth century? (2020) (a) Indian handicrafts were ruined. (b) Machines were introduced in the Indian textile industry in large numbers. (c) Railway lines were laid in many parts of the country. (d) Heavy duties were imposed on the imports of British manufactures.”,2020

A

5,a,”Declining and deindustrialisation of the Indian traditional cottage and handicraft industry was a direct effect of the industrial revolution in England along with the colonisation of India. Industrial Revolution and its impact on India’s indigenous industry: - The Industrial Revolution brought severe consequences to Indian society. - The fall and destruction of the urban and rural handicraft industry was one of the most significant effects of British Rule. - The Industrial Revolution in England also affected the textile industries in India in many ways, such as: English textiles were now posing a serious threat to Indian textiles in the European and American markets. Textiles from India were subject to high taxes in Britain. The African markets were successfully taken over by cheap, machine-made textiles from Britain. By the 1830s, inexpensive British manufactured products had inundated the Indian marketplaces. The Indian textile industries suffered as a result of them being less expensive than Indian textiles. - Following the Industrial Revolution, Britain began making textiles by a machine that were considerably more affordable and of higher quality than conventional Indian textiles. - The start of the industrial revolution had a negative impact on the Indian handicraft industry. - Additionally, the decline of the kings, chieftains, and zamindars who served as the employers of these masters of handicrafts led to the steady expansion of British dominion in India through outright battles with the rulers, which resulted in the suffering of handicrafts in India. - India became both a raw material supplier and a finished goods market. India, a country based on agriculture, became an economic colony. - The British items were inexpensive and only required a little amount of duty, while the Indian goods were subject to heavy import duties.”

How well did you know this?
1
Not at all
2
3
4
5
Perfectly
6
Q

6,”Which one of the following groups of plants was domesticated in the ‘New World’ and introduced into the ‘Old World’? (2019) (a) Tobacco, cocoa and rubber (b) Tobacco, cotton and rubber (c) Cotton, coffee and sugarcane (d) Rubber, coffee and wheat”,2019

A

6,a,”Tobacco, cocoa, and rubber are the groups of plants that were domesticated in the ‘New World’ and introduced into the ‘Old World’ through the European explorers in the late medieval or early modern era. The Columbian exchange, also known as the Columbian interchange, named for Christopher Columbus, was the widespread transfer of plants, animals, culture, human populations, technology, diseases, and ideas between the Americas, West Africa, and the Old World in the 15th and 16th centuries. It also relates to European colonization and trade following Christopher Columbus’s 1492 voyage. Invasive species, including communicable diseases, were a by-product of the Exchange. Old World to New World: Coffee, wheat, cotton, barley, rice, sugarcane and sugar beet, etc. New World to Old World: avocado, cashew, cocoa bean, potato, maize, rubber, tobacco, etc.”

How well did you know this?
1
Not at all
2
3
4
5
Perfectly
7
Q

7,”Economically, one of the results of the British rule in India in the 19th century was the: (2018) (a) increase in the export of Indian handicrafts (b) growth in the number of Indian owned factories (c) commercialization of Indian agriculture (d) rapid increase in the urban population”,2018

A

7,c,”One of the results of the British rule in India in the 19th century was the commercialization of agriculture by introducing cash crops like opium and indigo. Colonial rule has negatively impacted the Indian industries and handicrafts. The increase in the import of finished manufactured goods poses a threat to the indigenous factories. It was, economically, one of the results of British rule in India in the 19th Century was the growth of commercial crops like cotton, jute, groundnut, oilseeds, tobacco, sugarcane, etc. and the raw materials for the industries in Britain. There was no rapid increase in the urban population during 19th Century British India.”

How well did you know this?
1
Not at all
2
3
4
5
Perfectly
8
Q

8,”The staple commodities of export by the English East Indian Company from Bengal in the middle of the 18th century were (2018) (a) Raw cotton, oil-seeds and opium (b) Sugar, salt, zinc and lead (c) Copper, silver, gold, spices and tea (d) Cotton, silk, saltpeter and opium”,2018

A

8,d,”The East India Company was originally formed in Britain for pursuing trade with the East Indies in Southeast Asia. In fact, it ended up trading mainly with the Indian subcontinent and China, where the main items of trade were cotton, silk, tea, opium, and saltpetre (potassium nitrate).”

How well did you know this?
1
Not at all
2
3
4
5
Perfectly
9
Q

9,”Who among the following was/were associated with the introduction of Ryotwari Settlement in India during the British Rule? (2017) 1. Lord Cornwallis 2. Alexander Read 3. Thomas Munro Select the correct answer using the code given below: (a) 1 only (b) 1 and 3 only (c) 2 and 3 only (d) 1, 2 and 3”,2017

A

9,c,”Permanent Settlement: Started by Lord Cornwallis in 1793. Prevalent in Bengal, Bihar, Orissa, Varanasi (United Provinces), and Northern Karnataka. Covered 19% of British India. A new section of Zamindars was appointed who would provide 10/11th part of the revenue collected to the British while keeping the remaining. Ryotwari System: Started by Thomas Munro and Alexander Reed in 1820. Started in the Baramahal district of Tamil Nadu and later spread to Madras, parts of Bombay, East Bengal, Assam and Kurg (Karnataka). Covered 51% of British India. Ryots (farmers) were given ownership and other rights over the land and were required to pay the revenue directly to the government. Involved high taxes and strict collection. Mahalwari System: The Mahalwari system was introduced by Holt Mackenzie in 1822. Prevalent in United Provinces, Central Provinces and Punjab. Covered 30% of British India. The village or mahal served as the unit for collecting revenue. Village land belonged to the community and it was the responsibility of the entire community to pay the land revenue.”

How well did you know this?
1
Not at all
2
3
4
5
Perfectly
10
Q

10,”The Trade Disputes Act of 1929 provided for: (2017) (a) the participation of workers in the management of industries. (b) arbitrary powers to the management to quell industrial disputes. (c) an intervention by the British Court in the event of a trade dispute. (d) a system of tribunals and a ban on strikes.”,2017

A

10,d,”About the Trade Disputes Act, 1929: The Trade Disputes Act, 1929 was codified for five years as an experimental measure. The main objective of the Act was to make provisions for the establishment of Courts of Inquiry and Boards of Conciliation with a view to investigate and settle trade disputes. The Act banned unnoticed strikes or lockouts in the provision of public utility services. A strike or lockout that was intended to cause severe, widespread, or prolonged hardship for the community and compel the government to take or refrain from taking a particular course of action was also made illegal. This included strikes and lockouts that had any purpose other than to advance a trade dispute within the trade or industry in which the strikers or the employers locking out were engaged. The Act was amended in 1932 and was made permanent by the Trade Disputes (Extending) Act, 1934.”

How well did you know this?
1
Not at all
2
3
4
5
Perfectly
11
Q

11,”Consider the following statements: (2017) 1. The Factories Act, 1881 was passed with a view to fix the wages of industrial workers and to allow the workers to form trade unions. 2. N. M. Lokhande was a pioneer in organizing the labour movement in British India. Which of the above statements is/are correct? (a) 1 only (b) 2 only (c) Both 1 and 2 (d) Neither 1 nor 2”,2017

A

11,b,”The first Factory Act (1881) was passed in 1881 during the tenure of Lord Ripon. This legislation aimed to improve working conditions for factory workers. This act applies only to factories that use mechanical power, employing at least 100 workers. It prohibited employment of children under the age 7. Children between the ages 7-12 were to work for a maximum 9 hours. It also made compulsory that dangerous machinery should be fenced properly, but no mention of forming trade unions. Narayan Meghaji Lokhande (1848–1897) was the father of the trade union movement in India. He is remembered not only for ameliorating the working conditions of textile mill-hands in the 19th century but also for his courageous initiatives on caste and communal issues. Lokhande is acclaimed as the Father of the Trade Union Movement in India. Some of the rights mill workers got because of N M Lokhande were: Mill workers should get a weekly holiday on Sunday In the afternoon, workers should be entitled to half-hour recess. The mill should start working from 6:30 in the morning and close by sunset. The salaries of the workers should be given by the 15th of every month.”

How well did you know this?
1
Not at all
2
3
4
5
Perfectly
12
Q

12,”Who of the following was/were economic critic/critics of colonialism in India? (2015) 1. Dadabhai Naoroji 2. G. Subramania Iyer 3. R.C. Dutt Select the correct answer using the codes given below. (a) 1 only (b) 1 and 2 only (c) 2 and 3 only (d) 1, 2 and 3”,2015

A

12,d,”Dadbhai Naoroji, G.Subramania Iyer and R.C. Dutt are considered as the economic critics of colonialism in India. The early nationalist of India analysed the colonial character of British rule in India in great detail. The economic critique produced by them was one of the remarkable achievements of the moderate nationalists. Dadabhai Naoroji (Grand Old man of India): “Poverty and UnBritish Rule in India” - Propounded Drain of Wealth Theory. Uncovered the economic exploitation of India by the British. Romesh Chandra Dutta: “The Economic History of India” - Half of the annual GDP of India outflowed every year. G. V. Joshi: Pointed out that the development in India was like a subsidy to the British Industries. Other Critiques: Justice Mahadev Govind Ranade, G. Subramaniam Iyer, G.K. Gokhale etc. examined every facet of the economy and subjected the entire range of economic issues and colonial economic policies to minute scrutiny.”

How well did you know this?
1
Not at all
2
3
4
5
Perfectly
13
Q

13,”With reference to Ryotwari Settlement, consider the following statements: (2012) 1. The rent was paid directly by the peasants to the Government. 2. The government gave Pattas to the Ryots. 3. The lands were surveyed and assessed before being taxed. Which of the statements given above is/are correct? (a) 1 only (b) 1 and 2 only (c) 1, 2 and 3 (d) None”,2012

A

13,c,”The Ryotwari System was started by Thomas Munro and Alexander Reed. It was started in the Baramahal district of Tamil Nadu and later spread to Madras, parts of Bombay, East Bengal, Assam and Kurg (Karnataka). It covered 51% of British India. Ryots (farmers) were given the ownership and other rights (Pattas) over the land and were required to pay the revenue directly to the government. The Ryotwari System involved high taxes (50% in dryland and 60% in wetland).”

How well did you know this?
1
Not at all
2
3
4
5
Perfectly
14
Q

14,”Consider the following statements: (2012) The most effective contribution made by Dadabhai Naoroji to the cause of Indian National Movement was that he1. Exposed the economic exploitation of India by the British. 2. Interpreted the ancient Indian texts and restored the self-confidence of Indians. 3. Stressed the need for eradication of all the social evils before anything else. Which of the statements given above is/are correct? (a) 1 only (b) 2 and 3 only (c) 1 and 3 only (d) 1, 2 and 3”,2012

A

14,a,”Dadabhai Naoroji (Grand Old man of India) wrote the “Poverty and UnBritish Rule in India” a Propounded Drain of Wealth Theory and exposed the economic exploitation of India by the British. Swami Dayanand Saraswati, Swami Vivekananda, etc. interpreted the ancient texts and restored the self confidence of Indians, not Dadabhai Naoroji. Swami Dayananda Saraswati even gave the call to “go back to vedas”. Raja Rammohan Roy, Ishwar Chandra Vidyasagar, Keshav Chandra Sen, Atmaram Pandurang, etc. emphasised the need for eradication of all social evils before anything else. Dadabhai Naoroji wasn’t a social reformer.”

How well did you know this?
1
Not at all
2
3
4
5
Perfectly
15
Q

15,”Consider the following: (2012) 1. Assessment of land revenue on the basis of nature of the soil and the quality of crops. 2. Use of mobile cannons in warfare. 3. Cultivation of tobacco and red chillies. Which of the above was/were introduced into India by the English? (a) 1 only (b) 1 and 2 (c) 2 and 3 (d) None”,2012

A

15,d,”After getting the Diwani rights of Bengal, Bihar, and Orissa in 1765 the major aim of the British East India Company was to increase their land revenue collection. So its policies were aimed at getting maximum income from land without caring of its consequences on cultivators and peasants. Three major systems of land revenue collection existed in India - Zamindari, Ryotwari and Mahalwari. The first recorded use of Artillery in India is at the Battle of Adoni in 1368. In the Deccans the Bahamani kings led by Mohammand Shah Bahamani used a train of Artillery against the Raja of Vijaya-nagar. Tobacco and Red Chillies were introduced by Portuguese not British.”

How well did you know this?
1
Not at all
2
3
4
5
Perfectly
16
Q

16,”The tendency for increased litigation was visible after the introduction of the land settlement system of Lord Cornwallis in 1793. The reason for this is normally traced to which of the following provisions? (2011) (a) Making Zamindar’s position stronger vis-à-vis the ryot (b) Making East India Company an overlord of Zamindars. (c) Making judicial system more efficient (d) None of the (a), (b) and (c) above.”,2011

A

16,d,”Reforms of Cornwallis: The Permanent Land Revenue Settlement of Bengal, which was extended to the provinces of Bihar and Orissa, was Lord Cornwallis’ most notable administrative initiative. The reforms had increased the litigation after the introduction of the land settlement system and the main reason for this was the removal of Court Fee and now everybody could drag anybody to the courts. The extension to the right of appeal was also one of the reasons court fees were abolished by Cornwallis and Lawyers were prescribed their fees. Zamindari System: The Zamindari System was introduced by Lord Cornwallis in 1793 through the Permanent Settlement Act 1793. The land where the zamindars had previously been responsible for collecting revenue was given to them as ownership. The settlement is referred to as the Zamindari Settlement since it was made with the zamindars. Since they were regarded as the soil’s tillers, the ryots were made tenants. The demand for land revenue was predetermined and out of controlled revenue, 10/11th part was given to the company. Company also added the “Sunset clause” for the zamindars.”

How well did you know this?
1
Not at all
2
3
4
5
Perfectly
17
Q

17,”With reference to the period of colonial rule in India, “Home Charges” formed an important part of the drain of wealth from India. Which of the following funds constituted “Home Charges”? (2011) 1. Funds used to support the India Office in London. 2. Funds used to pay salaries and pensions of British personnel engaged in India. 3. Funds used for waging wars outside India by the British. Select the correct answer using the codes given below: (a) 1 only (b) 1 and 2 only (c) 2 and 3 only (d) 1, 2 and 3”,2011

A

17,b,”Drain of Wealth and Home Charges: Dadabhai Naoroji (Grand Old man of India) in his “Poverty and UnBritish Rule in India” (1901), propounded the Drain of Wealth Theory. Home Charges played a significant role in the Drain of Wealth from India. It comprised of: Funding for the Indian offices in London. Funds for paying the salaries and pensions of British employees working in India. Funds used for additional capital investments, such as railways, and to pay interest on debts. Statement 1 is correct: Funds used to support the India Office in London. Hence, Statement 2 is correct: Funds used to pay salaries and pensions of British personnel engaged in India.”

How well did you know this?
1
Not at all
2
3
4
5
Perfectly
18
Q

18,”Which of the following is/are the principal feature(s) of the Government of India Act, 1919? (2012) 1. Introduction of diarchy in the executive government of the provinces. 2. Introduction of separate communal electorates for Muslims. 3. Devolution of legislative authority by the centre to the provinces. Select the correct answer using the codes given below: (a) 1 only (b) 2 and 3 only (c) 1 and 3 only (d) 1, 2 and 3”,2012

A

18,c,”GoI Act 1919 introduced the dual scheme of governance known as ‘dyarchy’ in the executive government of the provinces. By demarcating and separating the central and provincial subjects, it relaxed the central control over the provinces. It introduced bicameralism and direct elections in the country. The devolution of legislative authority by the centre to the provinces was done in the Government of India Act, 1919. The Indian Council Act, 1909 or the Morley-Minto Reforms introduced separate communal electorates for Muslims. It was put into place to appease the moderates (in Congress) and introduces separate electorates based on religion. Therefore, Lord Minto came to be known as “Father of the Communal Electorate in India”.”

How well did you know this?
1
Not at all
2
3
4
5
Perfectly
19
Q

19,”The distribution of powers between the Centre and the States in the Indian Constitution is based on the scheme provided in the: (2012) (a) Morley-Minto Reforms, 1909 (b) Montagu-Chelmsford Act, 1919 (c) Government of India Act, 1935 (d) Indian Independence Act, 1947”,2012

A

19,c,”Government of India Act, 1935: Divided the powers between the Centre and provinces in terms of three lists– Federal List (for Centre, with 59 items), Provincial List (for provinces, with 54 items) and the Concurrent List (for both, with 36 items). Abolished dyarchy in the provinces and introduced ‘provincial autonomy’ in its place, while adopted the dyarchy at the Centre. Introduced bicameralism in six out of eleven provinces. Further extended the principle of communal representation to the depressed classes (Scheduled Castes), women and labour (workers). Abolished the Council of India, established by the Government of India Act of 1858.”

How well did you know this?
1
Not at all
2
3
4
5
Perfectly
20
Q

20,”What was the purpose for which Sir William Wedderburn and W.S. Caine had set up the Indian Parliamentary Committee in 1893? (2011) (a) To agitate for Indian political reforms in the House of Commons (b) To campaign for the entry of Indians into the Imperial Judiciary (c) To facilitate a discussion on India’s Independence in the British Parliament (d) To agitate for the entry of eminent Indians into the British Parliament”,2011

A

20,a,”Together with Dadabhai Naoroji and other supporters of India, Sir William Wedderburn and W. S. Caine established the “Indian Parliamentary Committee” in 1893. The Indian Parliamentary Committee’s mission is to lobby the House of Commons for political reforms in India. Sir William Wedderburn was a significant figure in the Indian National Congress and played a crucial role in advocating for political reforms in India. By a general understanding of modern history, we know congress was established in 1885 and 1893 was a phase of early moderates and demands were mostly concentrated about reforms.”

How well did you know this?
1
Not at all
2
3
4
5
Perfectly
21
Q

21,”The Ilbert Bill controversy was related to the: (2013) (a) Imposition of certain restrictions to carry arms by the Indians (b) Imposition of restrictions on newspapers and magazines published in Indian languages (c) Removal of disqualifications imposed on the Indian magistrates with regard to the trial of the Europeans (d) Removal of a duty on imported cotton cloth”,2013

A

21,c,”The Ilbert Bill Controversy 1883 was related to the removal of disqualifications imposed on the Indian magistrates with regard to the trial of the Europeans.”

How well did you know this?
1
Not at all
2
3
4
5
Perfectly
22
Q

22,”What was/were the object/objects of Queen Victoria’s Proclamation (1858)? (2014) 1. To disclaim any intention to annex the Indian States 2. To place the Indian administration under the British Crown. 3. To regulate East India Company’s trade with India Select the correct answer using the code given below. (a) 1 and 2 only (b) 2 only (c) 1 and 3 only (d) 1, 2 and 3”,2014

A

22,a,”The proclamation was read out by Lord Canning at a durbar held on November 1, 1858 at Allahabad. It put an end to the rule of the East India Company and the government of India came directly under the Crown. According to the proclamation Lord Canning became the first viceroy and governor general of India. The proclamation put an end to the era of further expansion of the British Empire in India.”

How well did you know this?
1
Not at all
2
3
4
5
Perfectly
23
Q

23,”The Government of India Act of 1919 clearly defined: (2015) (a) The separation of power between the judiciary and the legislature (b) The jurisdiction of the central and provincial governments (c) The powers of the Secretary of State for India and the Viceroy (d) None of the above”,2015

A

23,b,”The Queen Victoria’s Proclamation (1858) aimed at ending the rule of the company and transferring all the powers to the British crown after the 1857 revolt. It also promised non-interference in the religious affairs of the people, grant of equal protection of law and respect for ancient rights and customs of the people.”

How well did you know this?
1
Not at all
2
3
4
5
Perfectly
24
Q

24,”The Montague-Chelmsford Proposals were related to: (2016) (a) social reforms (b) educational reforms (c) reforms in police administration (d) constitutional reforms”,2016

A

24,d,”The GoI Act 1919 introduced a diarchy (rule of two individuals/parties) for the executive at the level of the provincial government. The diarchy was implemented in eight provinces: Assam, Bengal, Bihar and Orissa, Central Provinces, United Provinces, Bombay, Madras and Punjab. The Dyarchy system increased the authority of the provincial governments. In the province, the governor was to serve as the chief executive.”

How well did you know this?
1
Not at all
2
3
4
5
Perfectly
25
Q

25,”In the context of Indian history, the principle of ‘dyarchy (diarchy)’ refers to: (2017) (a) division of the central Legislature into two houses. (b) introductions of double Government i.e., central and Statement governments. (c) having two sets of rulers; One in London and another in Delhi. (d) division of the subjects delegated to the provinces into two categories.”,2017

A

25,d,”The GoI Act 1919 introduced a diarchy (rule of two individuals/parties) for the executive at the level of the provincial government. The diarchy was implemented in eight provinces: Assam, Bengal, Bihar and Orissa, Central Provinces, United Provinces, Bombay, Madras and Punjab. The Dyarchy system increased the authority of the provincial governments. In the province, the governor was to serve as the chief executive.”

How well did you know this?
1
Not at all
2
3
4
5
Perfectly
26
Q

26,”The object of the Butler Committee of 1927 was to: (2017) (a) Define the jurisdiction of the Central and Provincial Governments. (b) Define the powers of the Secretary of State for India. (c) Impose censorship on national press. (d) Improve the relationship between the Government of India and the Indian States.”,2017

A

26,d,”The Butler Committee was appointed under the chairmanship of Sir Harcourt Butler in 1927 to investigate and clarify the relationship between the paramount power of the British Raj in India, and the rulers of Princely States. There were two other members of the committee: William Searle Holdsworth and Sidney Peel. In the committee’s report of 1929, the ‘paramountcy’ doctrine was reaffirmed, and guidelines were given for its application, ensuring that the financial relationship between the Raj and the States should be fair. The relationship of the princely states with the British empire was not merely a contractual relationship, but a living, growing relationship shaped by the circumstances and policy, resting on the mixture of history and theory. British paramountcy was to stay intact to preserve the princely state. States should not be transferred without their own consent to a relationship with a new government in British India responsible to an Indian legislature.”

How well did you know this?
1
Not at all
2
3
4
5
Perfectly
27
Q

27,”In the Federation established by the Government of India Act of 1935, residuary powers were given to the: (2018) (a) Federal Legislature (b) Governor General (c) Provincial Legislature (d) Provincial Governors”,2018

A

27,b,”As per the Government of India Act of 1935, the Governor-General was empowered to authorize, either the Federal or the Provincial Legislature to enact a law with respect to any residuary matter. The Act divided the powers between the Centre and provinces into three lists: Federal List, Provincial List, and Concurrent List. The allocation of residuary powers was unique, vested neither in the central nor provincial legislatures, but the Governor-General had the authority to authorize either legislature to enact a law regarding any residuary matter. Additionally, the Act abolished dyarchy in the provinces, introduced provincial autonomy, enacted bicameralism in six out of eleven provinces, extended communal representation, and abolished the Council of India established by the Government of India Act of 1858.”

How well did you know this?
1
Not at all
2
3
4
5
Perfectly
28
Q

28,”Consider the following statements about ‘the Charter Act of 1813’: (2019) 1. It ended the trade monopoly of the East India Company in India except for trade in tea and trade with China. 2. It asserted the sovereignty of the British Crown over the Indian territories held by the Company. 3. The revenues of India were now controlled by the British Parliament. Which of the statements given above are correct? (a) 1 and 2 only (b) 2 and 3 only (c) 1 and 3 only (d) 1, 2 and 3”,2019

A

28,a,”The Charter Act of 1813 ended the trade monopoly of the East India Company except for the trade in tea, opium, and with China. It also brought the Company’s territories in India under the Sovereignty of the British Government. However, it did not control the revenue of India; that was done through Pitt’s India Act of 1784. Pitt’s India Act of 1784 extended the control of the British Government over the Company’s affairs and its administration in India, officially referring to the Company’s possessions in India as ‘British possessions in India’ for the first time and placing the Company’s activities and administration in India under the total control of the British Government.”

How well did you know this?
1
Not at all
2
3
4
5
Perfectly
29
Q

29,”Consider the following statements: (2021) 1. The Montagu-Chelmsford Reforms of 1919 recommended granting voting rights to all the women above the age of 21. 2. The Government of India Act of 1935 gave women reserved seats in the legislature. Which of the statements given above is/are correct? (a) 1 only (b) 2 only (c) Both 1 and 2 (d) Neither 1 nor 2”,2021

A

29,b,”The Montagu-Chelmsford Act, 1919 recommended several reforms including the introduction of dyarchy, division of provincial subjects into transferred and reserved, bicameralism, communal representation, establishment of a public service commission, and separated provincial budgets from the Central budget. The Government of India Act, 1935 further expanded on these reforms by dividing powers between the Centre and provinces, abolishing dyarchy, introducing provincial autonomy, extending communal representation, and abolishing the Council of India established by the Government of India Act of 1858.”

How well did you know this?
1
Not at all
2
3
4
5
Perfectly
30
Q

30,”Which amongst the following provided a common factor for tribal insurrection in India in the 19th century? (2011) (a) Introduction of a new system of land revenue and taxation of tribal products (b) Influence of foreign religious missionaries in tribal areas (c) Rise of a large number of money lenders, traders and revenue farmers as middlemen in tribal areas (d) The complete disruption of the old agrarian order of the tribal communities”,2011

A

30,d,”Option (d) provides a comprehensive explanation covering various causes of tribal revolts in India during the 19th century. It includes disruption of tribal life due to colonial rule, loss of land, imposition of restrictions on traditional practices, introduction of money lenders leading to exploitation, changes in land ownership, and social unrest due to missionary activities. Additionally, it mentions major tribal movements such as the Kol Rebellion, Santhal Rebellion, and Munda Rebellion. This explanation provides a broader understanding of the issues faced by tribal communities during colonial rule.”

How well did you know this?
1
Not at all
2
3
4
5
Perfectly
31
Q

31,”The demand for the Tebhaga Peasant Movement in Bengal was for: (2013) (a) the reduction of the share of the landlords from one-half of the crop to one third. (b) the grant of ownership of land to peasants as they were the actual cultivators of the land. (c) the uprooting of Zamindari system and the end of serfdom. (d) writing off all peasant debts.”,2013

A

31,a,”The Tebhaga Peasant Movement (1946) in Bengal demanded that the share of the landlords be reduced from one-half of the produce to one-third of the produce. The movement aimed to implement the Floud Commission recommendations, advocating for two-thirds’ share to the bargardars, or share-croppers, instead of the one-half share. The main slogan of the movement was “nij khamare dhan tolo,” meaning that sharecroppers should thresh their own paddy and not to the jotedar’s house as before, enforcing tebhaga. The movement, primarily centered in North Bengal, particularly among the Rajbanshis, saw significant participation from Muslims as well. However, it dissipated soon due to various factors, including the Bargardari Bill introduced by the League ministry, the Hindu Mahasabha’s campaign for a separate Bengal, and riots in Calcutta.”

How well did you know this?
1
Not at all
2
3
4
5
Perfectly
32
Q

32,”After the Santhal Uprising subsided, what was/ were the measure/measures taken by the colonial government? (2018) 1. The territories called ‘Santhal Paraganas’ were created. 2. It became illegal for a Santhal to transfer land to a non-Santhal. Select the correct answer using the code given below: (a) 1 only (b) 2 only (c) Both 1 and 2 (d) Neither 1 nor 2”,2018

A

32,c,”The measures taken by the Colonial Government as a result of the Santhal Uprising included the creation of a separate district of Santhal Parganas to pacify the Santhals. This action was aimed at separating the Santhal Parganas into its own district to address the grievances of the Santhals. Additionally, it became illegal for a Santhal to transfer land to a non-Santhal, as the British Government enacted laws to prevent the lands of tribals from being taken by outsiders.”

How well did you know this?
1
Not at all
2
3
4
5
Perfectly
33
Q

33,”Indigo cultivation in India declined by the beginning of the 20th century because of (2020) (a) peasant resistance to the oppressive conduct of planters (b) its unprofitability in the world market because of new inventions (c) national leaders’ opposition to the cultivation of indigo (d) Government control over the planters”,2020

A

33,b,”The decline in indigo cultivation in India by the beginning of the 20th century was primarily due to the unprofitability of indigo production in the world market because of new inventions of improved versions of chemical colors that substituted indigo. These inventions rendered indigo production unprofitable in the world market, leading to the decline in indigo cultivation. The Indigo Revolt of 1856-57 saw peasants refusing to produce indigo or sign contracts, resulting in a widespread strike that expanded to other parts of Bengal. The strike was supported by zamindars and intelligentsia, eventually leading to the establishment of a commission in favor of the peasants.”

How well did you know this?
1
Not at all
2
3
4
5
Perfectly
34
Q

34,”With reference to the history of India, “Ulgulan” or the Great Tumult is the description of which of the following events? (2020) (a) The Revolt of 1857 (b) The Mappila Rebellion of 1921 (c) The Indigo Revolt of 1859 – 60 (d) Birsa Munda’s Revolt of 1899-1900”,2020

A

34,d,”The Munda Rebellion, led by Birsa Munda in the years 1899–1900, aimed to establish Munda Raj or Munda authority in the Chotanagpur region. The rebellion was triggered by the replacement of the traditional Khuntkari system of Munda tribals with the zamindari system by the British in 1874. This change led to the monetization of the economy, forcing tribals to depend on cash for rent and daily needs, and rely on moneylenders who charged exorbitant rates. Birsa Munda organized masses to stop paying debts/interest and taxes, leading a revolt against Victorian rule and advocating for Munda Rule in Jharkhand. The rebellion aimed to address the grievances of the Munda tribals against the British colonial administration.”

How well did you know this?
1
Not at all
2
3
4
5
Perfectly
35
Q

35,”During the Indian freedom struggle, the National Social Conference was formed. What was the reason for its formation? (2012) (a) Different social reform groups or organizations of the Bengal region united to form a single body to discuss the issues of larger interest and to prepare appropriate petitions/representations to the government. (b) Indian National Congress did not want to include social reforms in its deliberations and decided to form a separate body for such a purpose. (c) Behramji Malabari and M.G. Ranade decided to bring together all the social reform groups of the country under one organization. (d) None of the statements (a), (b) and (c) given above is correct in this context.”,2012

A

35,b,”The Indian Social Conference (1887), formed by M. G. Ranade and Raghunath Rao in Madras, was a separate social reform organization from the Indian National Congress. It advocated for inter-caste marriage, opposed polygamy and kulinism, and launched the ‘Pledge Movement’ against child marriage. The conference focused on social reform issues and did not include them in the deliberations of the Indian National Congress.”

How well did you know this?
1
Not at all
2
3
4
5
Perfectly
36
Q

36,”Which of the following statements is/are correct regarding Brahmo Samaj? (2012) 1. It opposed idolatry. 2. It denied the need for a priestly class for interpreting the religious texts. 3. It popularized the doctrine that the Vedas are infallible. Select the correct answer using the codes given below: (a) 1 only (b) 1 and 2 only (c) 3 only (d) 1, 2 and 3”,2012

A

36,b,”The Brahmo Samaj, founded by Raja Ram Mohan Roy in 1828, opposed idolatry and denied the need for a priestly class for interpreting religious texts. It favored human reason and conscience, political upliftment of the masses, and widow remarriage. Brahmo Samaj vehemently opposed idol worship, caste system, untouchability, Sati, child marriage, and the Purdah system. While it did spread the religious treasure and teachings of the Vedas, it did not claim that Vedas are infallible.”

37
Q

37,”Satya Shodhak Samaj organized: (2016) (a) a movement for upliftment of tribals in Bihar (b) a temple-entry movement in Gujarat (c) an anti-caste movement in Maharashtra (d) a peasant movement in Punjab”,2016

A

37,c,”The Satyashodhak Samaj, founded by Jotirao Govindrao Phule in 1873, organized an anti-caste movement in Maharashtra. It opposed idolatry, condemned the caste system, and advocated for the spread of reason. The Samaj aimed to promote happiness, well-being, equality, and simplicity of religious practices for all people. Members of the Satyashodhak Samaj included people from various backgrounds, including Muslims, Brahmins, and government officials, with leading members and financial backers coming from Phule’s own Mali caste.”

38
Q

38,”Consider the following: (2016) 1. Calcutta Unitarian Committee 2. Tabernacle of New Dispensation 3. Indian Reform Association Keshab Chandra Sen is associated with the establishment of which of the above? (a) 1 and 3 only (b) 2 and 3 only (c) 3 only (d) 1, 2 and 3”,2016

A

38,b,”Keshab Chandra Sen, appointed as Acharya of the Brahmo Samaj in 1858, spread the ideas of the Samaj outside Bengal and showed radical views against the caste system, favoring inter-caste marriage. He founded the Brahmo Samaj of India in 1866 after being expelled from his position as Acharya. Additionally, he erected the Tabernacle of New Dispensation in honor of the Magha Festival and was instrumental in founding the Indian Reform Association in 1870, aimed at implementing concepts he encountered in Britain.”

39
Q

39,”In the context of Indian history, the Rakhmabai case of 1884 revolved around: (2020) 1. women’s right to gain education 2. age of consent 3. restitution of conjugal rights Select the correct answer using the code given below: (a) 1 and 2 only (b) 2 and 3 only (c) 1 and 3 only (d) 1, 2 and 3”,2020

A

39,b,”Dr. Rakhmabai, an Indian physician and feminist, was involved in a landmark legal case regarding her marriage as a child bride. Married at eleven years old, she faced a legal battle initiated by her husband, Dadaji Bhikaji, seeking ““restitution of conjugal rights”” after twelve years of marriage. The case led to the development of the Age of Consent Act in 1891. Behramji Malabari and Pandita Ramabai established the Rakhmabai Defense Committee to support her during the case.”

40
Q

40,”Consider the following freedom fighters: (2022) 1. Barindra Kumar Ghosh 2. Jogesh Chandra Chatterjee 3. Rash Behari Bose Who of the above was/were actively associated with the Ghadar Party? (a) 1 and 2 (b) 2 only (c) 1 and 3 (d) 3 only”,2022

A

40,d,”The Ghadar Party, formed in 1913, was a revolutionary group organized around the weekly newspaper ““The Ghadar,”” headquartered in San Francisco with branches along the US coast and in the Far East. It included ex-soldiers and peasants from Punjab who migrated to the USA and Canada for better employment opportunities. The party aimed to organize assassinations of officials, publish revolutionary literature, work among Indian troops abroad, procure arms, and incite a revolt in British colonies. Prominent leaders included Lala Hardayal, Ramchandra, Bhagwan Singh, Kartar Singh Saraba, and Barkatullah.”

41
Q

41,”What was the reason for Mahatma Gandhi to organise a satyagraha on behalf of the peasants of Kheda? (2011) 1. The Administration did not suspend the land revenue collection in spite of a drought. 2. The Administration proposed to introduce Permanent Settlement in Gujarat. Which of the statements given above is/are correct? (a) 1 only (b) 2 only (c) Both 1 and 2 (d) Neither 1 nor 2”,2011

A

41,a,”Kheda Satyagraha (1918) was the first Non-cooperation Movement, organized due to crop failure in Kheda district of Gujarat. Farmers, entitled to remission according to revenue code, were demanded taxes by the government despite low yield. Mahatma Gandhi led the satyagraha, supported by leaders like Sardar Vallabhbhai Patel, Mohanlal Pandya, and Narhari Parekh, resulting in the return of confiscated property, reduced tax increases, and suspended taxes for the year. Permanent Settlement, introduced by Lord Cornwallis in 1793, applied to Bengal, Bihar, Orissa, Varanasi (United Provinces), and Northern Karnataka, not Gujarat.”

42
Q

42,”The Partition of Bengal made by Lord Curzon in 1905 lasted until: (2014) (a) The First World War when Indian troops were needed by the British and the partition was ended. (b) King George V abrogated Curzon’s Act as the Royal Durbar in Delhi in 1911 (c) Gandhiji launched his Civil Disobedience Movement. (d) The Partition of India in 1947 when East Bengal became East Pakistan.”,2014

A

42,b,”The Partition of Bengal (1905) was announced in 1903 and implemented in October 1905, aiming to divide Bengal into eastern and western regions for administrative convenience and Assam’s development. Moderates like Surendranath Banerjee and K.K Mitra led anti-partition campaigns through newspapers like Hitabadi and Sanjibani. The annulment of the partition in 1911 during the visit of King George V and Queen Mary aimed to curb rising revolutionary extremism, disappointing Muslim political elites.”

43
Q

43,”The Ghadr (Ghadar) was a: (2014) (a) revolutionary association of Indians with headquarters at San Francisco. (b) nationalist organisation operating from Singapore. (c) militant organisation with headquarters at Berlin (d) Communist movement for India’s freedom with head-quarters at Tashkent.”,2014

A

43,a,”The Ghadar Party, formed around the newspaper ““The Ghadar,”” aimed at revolutionary activities abroad, led by figures like Lala Hardayal, Ramchandra, Bhagwan Singh, Kartar Singh Saraba, Barkatullah, and Bhai Parmanand. Pre-Ghadar activities involved Ramdas Puri, G.D. Kumar, Taraknath Das, Sohan Singh Bhakna, and Lala Hardayal. The party planned assassinations, disseminated anti-imperialist literature, coordinated with Indian troops abroad, and aimed to incite a simultaneous uprising in British colonies.”

44
Q

44,”Which one of the following movements has contributed to a split in the Indian National Congress resulting in the emergence of ‘moderates’ and ‘extremists’? (2015) (a) Swadeshi Movement (b) Quit India Movement (c) Non-Cooperation Movement (d) Civil Disobedience Movement”,2015

A

44,a,”The Swadeshi Movement, triggered by the Bengal Partition, spread nationally and contributed to the split in the Indian National Congress in 1907 during the Surat Session. Extremists like Bal Gangadhar Tilak and Lala Lajpat Rai sought presidency, while Moderates proposed Rashbehari Ghosh to exclude Tilak. The session, known as the Surat Split, highlighted differences over swadeshi, boycott, and national education, leading to a divide between Moderates and Extremists within the Congress.”

45
Q

45,”What was the main reason for the split in the Indian National Congress at Surat in 1907? (2016) (a) Introduction of communalism into Indian politics by Lord Minto (b) Extremists’ lack of faith in the capacity of the moderates to negotiate with the British Government (c) Foundation of Muslim League (d) Aurobindo Ghosh’s inability to the elected as the President of the Indian National Congress”,2016

A

45,b,”Option (b) is correct: On the backdrop of the Swadeshi Movement, the Extremists wanted either B.G. Tilak or Lajpat Rai as President. Moderates proposed Rashbehari Ghosh and wanted the session to be in Surat in order to exclude Tilak from the presidency. Since a leader from the host province could not be session president, they also sought to drop resolutions on swadeshi, boycott and national education. Thus it resulted in a split in the congress. Therefore, Extremists’ lack of faith in the capacity of the moderates to negotiate with the British Government’.”

46
Q

46,”The ‘Swadeshi’ and ‘Boycott’ were adopted as methods of struggle for the first time during the: (2016) (a) agitation against the Partition of Bengal (b) Home Rule Movement (c) Non-Cooperation Movement (d) visit of the Simon Commission to India”,2016

A

46, (a), Option (a) is correct: The ‘Swadeshi’ and ‘Boycott’ were the mass movements to oppose the Bengal partition. Option (b) is incorrect: The Home Rule Movement in 1916 was the Indian response to the First World War in a less charged but in more effective way. With people already feeling the burden of war time miseries caused by high taxation and a rise in prices, Tilak and Annie Besant ready to assume the leadership, the movement started with great vigour. Option (c) is incorrect: The time period of Swadeshi and Boycott Movement and Non-Cooperation Movement is different. Therefore it has no correlation with the Swadeshi and Boycott Movement. Option (d) is incorrect: Simon Commission visited in 1928 to review the constitutional reform in Britain’s largest and most important possession.

47
Q

47,”Which one of the following is a very significant aspect of the Champaran Satyagraha? (2018) (a) Active all-India participation of lawyers, students and women in the National Movement (b) Active involvement of Dalit and Tribal communities of India in the National Movement (c) Joining of peasant unrest to India’s National Movement (d) Drastic decrease in the cultivation of plantation crops and commercial crops”,2018

A

47, (c), Champaran Satyagraha (1917): Mahatma Gandhi’s initial experiment of satyagraha was known as Champaran Satyagraha. It was undertaken after Mahatma Gandhi learned about the abuses suffered by farmers, who were forced into growing indigo by British planters and estate owners. The tenants from Champaran were forced under the law to grow indigo on 3/20th part of the land (Tinkathia System) for his landlord. To increase their profits, European planters demanded exorbitant rents and illegitimate fees. Rajkumar Shukla invited Gandhi to look into the problems of Indigo planters in Champaran, Bihar. Gandhiji intended to conduct a thorough investigation in the district and demand action based on its results. During this time Gandhiji was even charged with violating law and was told to leave Champaran, but he refused to leave. Later, the then Lieutenant Governor of Bihar ordered the withdrawal of the case against Gandhi, and the Collector wrote to Gandhi saying he was free to conduct the inquiry. In the history of the independence struggle, this tiny step in the form of passive protest was a great leap, signalling the beginning of the Gandhian period. Leaders associated: Rajendra Prasad, Mazhar-ul-Haq, Mahadeo Desai, Narhari Parekh, J.B. Kripalani etc. Gandhi’s position in India’s resistance to the British raj was solidified by the victory at Champaran. NOTE: in given statements, notice “all-India participation” in statement (a) and “Drastic decrease” in statement (d), we can take risk of eliminating these extreme statements. Always remember, there is no set rule for this “extreme word” elimination method. There are also some instances where sentences with extreme words were correct, for instance a question in 2017 about Foreign exchange of India. In the mentioned question, the statement with “drastic increase” was correct.

48
Q

48,”With reference to Swadeshi Movement consider the following statements: (2019) 1. It contributed to the revival of the indigenous artisan crafts and industries. 2. The National Council of Education was established as a part of the Swadeshi Movement. Which of the statements given above is/are correct? (a) 1 only (b) 2 only (c) Both 1 and 2 (d) Neither 1 nor 2”,2019

A

48, (c), Swadeshi and Boycott Movements: There were large-scale protests against the partition of Bengal. Later, the movement spread to other regions of the nation: The Boycott Resolution was passed at the Calcutta Town Hall on August 7, 1905, formally launching the Swadeshi Movement. Partition came into force: October 16, 1905 There were also new means of protest like processions, public meetings, and boycotts of imported goods. Imaginative use of Traditional Festivals, Melas: Tilak’s Ganapati and Shivaji festivals became a medium of swadeshi propaganda. Statement 1 is correct: Importance to Self-Reliance (Atma Shakti): Emphasis was placed on honour, social and economic regeneration of the villages. Swadeshi enterprises: Swadeshi textile mills, banks, etc, were set up. At Tuticorin, V.O. Chidambaram Pillai founded the Swadeshi Steam Navigation Company. Statement 2 is correct: Swadeshi programme or National Education: 1. Bengal National College - Aurobindo Ghosh as its Principal - inspired by Tagore’s Shantiniketan. 2. The National Council of Education (1906) was set up to organise national education in the vernacular medium. 3. The Bengal Institute of Technology was established to provide technical training. Cultural Impact: Rabindranath Tagore wrote Amar Sonar Bangla, Subramania Bharati wrote Swadesha Geetham. Corps of Volunteers or ‘Samitis’: they generated political consciousness among the masses. NOTE: Question on similar theme/topic i.e Surat Split/Partition of Bengal/Swadeshi Movement came in 2014 (on annulment of partition of Bengal), 2015 and 2016 (total 2 questions). Hereby, revising PYQ stands important.

49
Q

49,”With reference to the period of Indian freedom struggle, which of the following was/were recommended by the Nehru report? (2011) 1. Complete Independence for India. 2. Joint electorates for reservation of seats for minorities 3. Provision of fundamental right for the people of India in the constitution. Select the correct answer using the codes given below: (a) 1 only (b) 2 and 3 only (c) 1 and 3 only (d) 1, 2 and 3”,2011

A

49, (b), Nehru Report (1928): As an answer to Lord Birkenhead’s challenge, the Nehru report was prepared by a committee headed by Motilal Nehru, the committee included Tej Bahadur Sapru, Subhash Bose, M.S. Aney, Mangal Singh, Ali Imam, Shuab Qureshi and G.R. Pradhan as its members. Recommendations:  Dominion status on lines of self-governing dominions. Therefore, statement 1 is incorrect.  Rejection of separate electorates. Joint electorates with reservation of seats for Muslims at the Centre and in provinces where they were in minority. Therefore, statement 2 is correct.  Linguistic provinces.  Nineteen fundamental rights, including the rights to form unions, the right to equal treatment for women, and universal adult suffrage. Therefore, statement 3 is correct.  Responsible government at the Centre and in provinces.  Full protection of the cultural and religious interests of Muslims.  Complete dissociation of State from religion.  The three Indian Round Table Conference (1930–1932) attendees had access to both the Nehru Report and the Simon Commission report.  Nehru and Subash Bose rejected the congress goal and set up the Independence for India league. NOTE: If we read NCERT of Modern India, we will come to know “Dominion Status” for India was the principal demand of the Nehru report. Just knowing this basic fact, we can eliminate option 1 and you will get the answer.

50
Q

50,”The Rowlatt Act aimed at: (2012) (a) Compulsory economic support to war efforts (b) Imprisonment without trial and summary procedures for trial (c) Suppression of the Khilafat Movement. (d) Imposition of restrictions on freedom of the press.”,2012

A

50, (b), Option (a) is incorrect: The drain on the Indian economy in the form of cash, kind and loans to the British government came to about 367 million pounds during the WW-I. Rowlatt Act has no relation with draining Indian money or economic support to war efforts. Option (b) is correct: Anarchical and Revolutionary Crimes Act/ Rowlatt act gives the notorious power to imprison activists without trial for two years, even possession of seditious newspapers is adequate evidence of guilt. Option (c) is incorrect: The Khilafat Movement (1919-20) was a movement to express Muslim support for the Caliph of Turkey, to protect the institution of the Khalifa in Turkey, against the allied powers, particularly Britain. The Rowlatt Act was not aimed at suppressing the Khilafat Movement. Option (d) is incorrect: Vernacular Press Act, 1878, The Newspaper Act, 1908 and The Indian Press Act, 1931 were passed in order to impose restrictions

51
Q

51,”The Lahore Session of the Indian National Congress (1929) is very important in history, because: (2012) 1. The Congress passed a resolution demanding complete independence. 2. The rift between the extremists and moderates was resolved in that Session. 3. A resolution was passed rejecting the two-nation theory in that Session Which of the statements given above is/are correct? (a) 1 only (b) 2 and 3 (c) 1 and 3 (d) None of the above”,2012

A

51, (a), Statement 1 is correct: In the Lahore Session of INC(1929), the resolution on ‘Poorna Swaraj’ or complete independence was passed. Statement 2 is incorrect: The rift between the extremists and moderates took place in the Surat Session(1907) and the split was resolved in the Lucknow Session 1916. Statement 3 is incorrect: The Cabinet Mission recommended an undivided India and rejected the demand of the Muslim League for a separate Pakistan. The Cabinet Mission felt that a separate state would contain a larger proportion of the Non-Muslim population and a sizable population of Muslims will be left in India, if two states are declared.

52
Q

52,”The people of India agitated against the arrival of the Simon Commission because: (2013) (a) Indians never wanted the review of the working of the Act of 1919 (b) Simon Commission recommended the abolition of Dyarchy (Diarchy) in the Provinces (c) There was no Indian member in the Simon Commission (d) The Simon Commission suggested the partition of the country”,2013

A

52, (c), Simon Commission (1927): The Indian Statutory Commission, commonly referred to as the Simon Commission, was a group of seven British Members of Parliament under the chairmanship of Sir John Simon who arrived in British India in 1928. Objective: The Simon Commission was appointed by the British government to assess India’s constitutional development and recommend constitutional reforms. Many Indians strongly opposed the Commission for a variety of reasons. The main reason behind the agitation against Simon Commission was that there were no Indian members of the panel and only seven British members of the British Parliament. This was viewed as racist and colonialist.

53
Q

53,”The 1929 Session of Indian National Congress is on significance in the history of the Freedom Movement because the: (2014) (a) attainment of Self-Government was declared as the objective of the Congress. (b) Attainment of Poorna Swaraj was adopted as the goal of the Congress. (c) Non-Cooperation Movement was launched. (d) Decision to participate in the Round Table Conference in London was taken.”,2014

A

53, (b), Option (a) is incorrect: Attainment of Self-Government was declared as the objective of the Congress in the Calcutta Session, 1906. Option (b) is correct: Attainment of Poorna Swaraj was adopted as the goal of the Congress in the Lahore Session, 1929 under the presidentship of Jawaharlal Nehru. Along with this, the Congress made the decision to abstain from the First Round Table Conference because the British Government declined to call a Constituent Assembly to draft a constitution for India. Launch of a civil disobedience movement for complete independence and 26 January to be observed as ‘Independence Day’/Swarajya Day were also decided in the Lahore Session. Option (c) is incorrect: Mahatma Gandhi moved the Non-cooperation resolution in the Special Calcutta Session of Congress in 1920. Option (d) is incorrect: Decision to participate in the Round Table Conference in London was taken in the Karachi Session of Congress in 1931.

54
Q

54,”With reference to Rowlatt Satyagraha, which of the following statements is/are correct ? (2015) 1. The Rowlatt Act was based on the recommendations of the ‘Sedition Committee’. 2. In Rowlatt Satyagraha, Gandhiji tried to utilize the Home Rule League. 3. Demonstrations against the arrival of the Simon Commission coincided with Rowlatt Satyagraha. Select the correct answer using the codes given below. (a) 1 only (b) 1 and 2 only (c) 2 and 3 only (d) 1, 2 and 3”,2015

A

54, (b), Statement 1 is correct: Rowlatt Act was passed on the recommendations of the Sedition Committee chaired by Sir Sidney Rowlatt. Statement 2 is correct: In organising his satyagraha, Gandhiji tried to utilise three types of political networks: The Home Rule Leagues; Certain Pan-Islamist groups and Satyagraha Sabha. Statement 3 is incorrect: The two time periods are not the same. It has a gap of 9 years, the incident of Rowlatt Satyagraha took place in 1919 and the Demonstrations against the arrival of Simon Commission took place in 1928, thus the two events cannot coincide.

55
Q

55,”In 1920, which of the following changed its name to “Swarajya Sabha”? (2018) (a) All India Home Rule League (b) Hindu Mahasabha (c) South Indian Liberal Federation (d) The Servants of India Society”,2018

A

55, (a), Home Rule League Movement (1916): It was an Indian response to World War - I, inspired by Irish Home Rule Leagues. The movement sought to achieve self-rule through political discussion and education, public gatherings, etc. The government responded with severe repression, but it prepared masses for Gandhian style of politics. Tilak launched the Indian Home Rule League in April 1916 at Belgaum. Annie Besant launched the Home Rule League in September 1916 at Madras. The All India Home Rule League became Swarajya Sabha in 1920. Its demands included Swarajya, formation of linguistic states and education in the vernacular. Leaders: Motilal Nehru, Lala Lajpat Rai, Jawaharlal Nehru, Bhulabhai Desai, Chittaranjan Das, Madan Mohan Malaviya, Mohammad Ali Jinnah, Tej Bahadur Sapru. Anglo-Indians, most of the Muslims and Non brahmins from south did not join as they felt it would mean home rule of the Hindu majority.

56
Q

56,”With reference to the proposals of Cripps Mission, consider the following statements: (2022) 1. The Constituent Assembly would have members nominated by the Provincial Assemblies as well as the Princely States. 2. Any Province, which is not prepared to accept the new Constitution would have the right to sign a separate agreement with Britain regarding its future status. Which of the statements given above is/are correct? (a) 1 only (b) 2 only (c) Both 1 and 2 (d) Neither 1 nor 2”,2022

A

56, (b), Cripps Mission (1942): In March 1942, a mission headed by Stafford Cripps was sent to India with constitutional proposals to seek Indian support for World War II. Because of the reverses suffered by Britain in South-East Asia, the Japanese threat to invade India seemed real now and Indian support became crucial. There was pressure on Britain from the Allies (USA, USSR, and China) to seek Indian cooperation. Provisions: Indian union with a Dominion status would be set up and it would be free to decide its relation with the UN and the Commonwealth. After the war the Constituent Assembly would be formed with members from Provinces (elected through proportional representation) and Princely states (nominated). (Hence, statement 1 is incorrect.) Conditions of Accepting New Constitution: Any Province not willing to join the Union can have a separate Constitution and form a Separate union. (Hence, statement 2 is correct.) New Constitution making Body and the Government would negotiate a treaty for Transfer of Power.

57
Q

57,”Which one of the following observations is not true about the Quit India Movement of 1942? (2011) (a) It was a non-violent movement (b) It was led by Mahatma Gandhi (c) It was a spontaneous movement (d) It did not attract the labour class in general”,2011

A

57, (b), Quit India Movement/August Revolution (1942): After failure of Cripps’s Mission, a Resolution was framed by Gandhiji for British withdrawal and non-violent Non-Cooperation Movement against Japanese Invasion. Resolution was accepted in the CWC meeting in Wardha on July 14, 1942. AICC Gowalia Tank Meeting, Bombay: The Quit India Resolution was ratified in a Congress meeting at Gowalia Tank on 8th August 1942. Apart from QIM, other resolutions involved the Civil Disobedience Movement against British Rule. Mantra of “Do or Die” was adopted in this Session by Gandhiji. Major activity after QIM resolution was passed: It was started in response to Mahatma Gandhi’s national appeal for satyagraha. Gandhiji advocated for “an orderly British withdrawal” from India as the All-India Congress Committee announced a nationwide protest. Destruction of symbols of Public Authority. Underground activities: Usha Sharma started an underground Radio to give fuel to the movement. Yusuf Meherally, a communist and trade unionist who also held the office of Mayor of Mumbai, is credited with coining the phrase “Quit India.” Parallel Government: established in Ballia (UP), Tamluk (Bengal) organised “Bidyut Bahini”, Satara (Maharashtra) “Prati Sarkar” was organised by Y. B. Chavan, Nana Patil etc. Youth, Women, Workers, Peasants, Government officials especially of lower levels, Muslims, Communists participated in the movement. Upper class remained Loyalists. There were no communal clashes during the movement. But there was severe government repression with upto 10,000 being killed. Option (a) is correct: The movement basically promoted the non-violent and non-cooperative movement. Option (b) is incorrect: The British Govt responded to the call of Gandhi by arresting all major Congress leaders the very next day. Gandhi, Nehru, Patel, etc. were all behind the bars. This left the movement in the hands of the younger leaders like Jayaprakash Narayan and Ram Manohar Lohia, besides many regional leaders also rose up to lead this movement in their respective regions. Option (c) is correct: The Quit India movement was the spontaneous participation of the masses compared to the other Gandhian movements like non-cooperation and civil disobedience. Option (d) is correct: Quit India Movement did not attract the labour class in general. NOTE: Here we have to identify the option which is not true about the QIM.

58
Q

58,”With reference to Indian freedom struggle, Usha Mehta is well-known for: (2011) (a) Running the secret Congress Radio in the wake of Quit India Movement (b) Participating in the second round Table Conference (c) Leading a contingent of Indian National Army (d) Assisting in the formation of Interim Government under Pandit Jawaharlal Nehru”,2011

A

58, (a), Option (a) is correct: Usha Sharma started an underground Radio to give fuel to the movement. Hence, she was well-known for underground activities during the Quit India Movement. Option (b) is incorrect: Second Round Table Conference was held in London between September 7, 1931 to December 1, 1931 and Sarojini Naidu participated to represent women. Option (c) is incorrect: Usha Mehta was part of a small group which ran the Congress radio. She started an Underground Radio in Bombay whereas Captain Lakshmi Sahgal was the commander of the Jhansi Regiment of INA. Option (d) is incorrect: She was not a part of the Interim Government. NOTE: It is always advisable to read in detail about the personalities involved in various movements. If we observe PYQs carefully, in every alternate year, there are personality based questions.

59
Q

59,”The Congress ministries resigned in the seven provinces in 1939, because: (2012) (a) The Congress could not form ministries in the other four provinces. (b) Emergence of a ‘left wing’ in the Congress made the working of the ministries impossible. (c) There were widespread communal disturbances in their provinces. (d) None of the statements (a), (b) and (c) given above is correct.”,2012

A

59, (d), The Congress Ministries: In the early 1937, elections to provincial assemblies were held and Congress Ministries were formed in Bombay, Madras, Central Provinces, United Provinces, Bihar, Orissa, Assam, and NWFP. By 1939, there were internal strifes, opportunism and hunger for power had started surfacing among Congressmen, yet they were able to utilise the council work to their advantage to a great extent and ruled for 28 months. Resignation of Congress Ministries (1939): The Congress Ministries resigned in protest because the then Viceroy Lord Linlithgow (1936-1944) action of declared India to be belligerent in WW-II without even consulting the Indian people.

60
Q

60,”Quit India Movement was launched in response to: (2013) (a) Cabinet Mission Plan (b) Cripps Proposals (c) Simon Commission Report (d) Wavell Plan”,2013

A

60, (b), The failure of the Cripps Mission to resolve the constitutional deadlock made it clear to the nationalists that any silence would amount to accepting the right of the British to decide the fate of Indians without consulting them. Quit India Movement was launched in response to Mahatma Gandhi’s national call for satyagraha. The All-India Congress Committee proclaimed a mass protest demanding what Gandhiji called “an orderly British withdrawal” from India. The Quit India resolution was ratified at the Congress meeting at Gowalia Tank, Bombay on August 8, 1942.

61
Q

61,”The Radcliffe Committee was appointed to: (2014) (a) Solve the problem of minorities in India (b) Given effect to the Independence Bill (c) Delimit the boundaries between India and Pakistan (d) Enquire into the riots in East Bengal”,2014

A

61, (c), In the event of Partition, a boundary commission led by Sir Cyril Radcliffe was established which was to delimit the boundaries between India and Pakistan. Boundary Commission, consultative committee created in July 1947 to recommend how the Punjab and Bengal regions of the Indian subcontinent were to be divided between India and Pakistan shortly before each was to become independent from Britain.

62
Q

62,”With reference to Congress Socialist Party, consider the following statements: (2015) 1. It advocated the boycott of British goods and evasion of taxes. 2. It wanted to establish the dictatorship of the proletariat. 3. It advocated separate electorate for minorities and oppressed classes. Which of the statements given above is/are correct? (a) 1 and 2 only (b) 3 only (c) 1, 2 and 3 (d) None”,2015

A

62, (d), Statement 1 is incorrect: On May 17, 1934, at Patna, with Acharya Narendra Dev as president and Jayaprakash Narayan (JP) as general secretary, the Congress Socialist Party (CSP) was founded within the Congress. The CSP advocated decentralised socialism in which co-operatives, trade unions, independent farmers, and local authorities would hold a substantial share of the economic power. It did not advocate dictatorship of the proletariat. Statement 2 is incorrect: Their focus was to attain independence and believed in socialism through nationalism. The CSP promoted a decentralised socialist system in which local authorities, cooperatives, trade unions, and independent farmers would control a sizable portion of the economic power. Hence, it did not aim to establish the dictatorship of the proletariat. Statement 3 is incorrect: As secularists, they hoped to transcend communal divisions through class solidarity. Many people, like Narendra Deva and Basawon Singh (Sinha), favoured a democratic socialism that was separate from both reformist social democracy and Marxism.

63
Q

63,”Who of the following organised a march on the Tanjore coast to break the Salt Law in April 1930? (2015) (a) V.O. Chidambaram Pillai (b) C. Rajagopalachari (c) K. Kamaraj (d) Annie Besant”,2015

A

63, (b), Civil Disobedience Movement - The Salt Satyagraha and other upsurges: Dandi March (March 12- April 6, 1930): The Dandi march was led by Gandhi and began at Sabarmati Ashram and ended close to the Dandi village’s coastal location; it was also known as Salt Satyagraha. He reached Dandi on April 6, 1930, and broke the salt law by collecting salt from the beach. Satyagraha at Different Places: Tamil Nadu: C Rajagopalachari organised a march from Tiruchirapalli to Vedaranniyam on Tanjore coast. Malabar: K. Kelappan known for Vailkom Satyagraha organised salt marches. Orissa: Gopal Bandhu Chaudhuri organised in Balasore, Cuttack etc. Bihar: Non chowkidari tax was imposed. Peshawar: Badshah khan/khan Abdul Gaffar Khan(Frontier Gandhi) who organised Khudaikhidmatgars (red shirts) organised a powerful march. Dharasana: Sarojini Naidu led the campaign. Nagaland: Rani Gaidinliu at the age of 13years raised the banner against the British.

64
Q

64,”With reference to the Cabinet Mission, which of the following statements is/are correct? (2015) 1. It recommended a federal government. 2. It enlarged the powers of the Indian courts. 3. It provided for more Indians in the ICS. Select the correct answer using the code given below. (a) 1 only (b) 2 and 3 (c) 1 and 3 (d) None of these”,2015

A

64, (a), Statement 1 is correct: On 22nd January 1946, the decision to send Cabinet Mission was taken and on 19th February 1946, the British PM C.R Attlee Government announced in the House of Lords about the mission and the plan to quit India. A high-powered mission of three British Cabinet members- Lord Pethick-Lawrence(the Secretary of State for India), Sir Stafford Cripps(President of the Board of Trade) and A. V. Alexander(the First Lord of the Admiralty) reached Delhi on 24th March 1946. All subjects other than Union subjects and residuary power would vest in the provinces of British India. Thus, the Cabinet Mission plan proposed a weak Centre with Provincial autonomy, essentially proposing a federal structure of government. Statements 2 and 3 are incorrect: The plan did not include the enlargement of the powers of the Indian Court nor provided for more Indians in the ICS.

65
Q

65,”The plan of Sir Stafford Cripps envisaged that after the Second World War: (2016) (a) India should be granted complete independence (b) India should be partitioned into two before granting independence (c) India should be made a republic with the condition that she will join the Commonwealth (d) India should be given Dominion status”,2016

A

65, (d), Cripps Mission (1942): In March 1942, a mission headed by Stafford Cripps was sent to India with constitutional proposals to seek Indian support for World War II. Proposals: Indian union with a Dominion status would be set up and it would be free to decide its relation with the UN and the Commonwealth. After the war, the Constituent Assembly would be formed with members from Provinces (elected through proportional representation) and Princely states (nominated). Conditions of accepting the New Constitution: Any Province not willing to join the Union can have a separate Constitution and form a separate union. New Constitution-making Body and the Government would negotiate a treaty for Transfer of Power.

66
Q

66,”With reference to Indian freedom struggle, consider the following events: (2017) 1. Mutiny in Royal Indian Navy 2. Quit India Movement launched 3. Second Round Table Conference What is the correct chronological sequence of the above events? (a) 1-2-3 (b) 2-1-3 (c) 3-2-1 (d) 3-1-2”,2017

A

66, (c), Second Round Table Conference (1931): In London, between September 7, 1931 to December 1, 1931. Result of the Gandhi-Irwin Pact. Gandhi was the sole representative of the INC. Madan Mohan Malviya and A. Rangaswami Iyengar were also there. Muslim league represented by Aga Khan III, Maulana Mohammad Jinnah. Depressed classes by B. R. Ambedkar, Rettamalai Srinivasan, Sardar Sampuran Singh etc. Sarojini Naidu participated to represent women. Lord Willingdon was the viceroy of India. A Communal Award for representing minorities in India by providing for separate electorates for minority communities. Gandhi was against this. The government refused Indian demands and talks failed. Quit India Movement/August Revolution (1942): After the failure of Cripps’s Mission, a Resolution was framed by Gandhiji for British withdrawal and non-violent Non-Cooperation Movement against the Japanese Invasion. The resolution was accepted in the CWC meeting in Wardha on July 14, 1942. Royal Indian Naval Mutiny (1946): On February 18, 1946, a mutiny started among the ratings of HMIS Talwar due to poor pay, inadequate food, and racial discrimination. The Muslim League, Congress, and CPI’s red flags were raised. The intervention of Sardar Patel put an end to the rebellion. The mutineers faced trials despite promises. This led to Violence in Bombay, with the loss of 200 lives. Significance: On February 19, 1946, a Cabinet Mission was sent to India.

67
Q

67,”With reference to the British colonial rule in India, consider the following statements: (2019) 1. Mahatma Gandhi was instrumental in the abolition of the system of ‘indentured labour’. 2. In Lord Chelmsford’s War Conference’, Mahatma Gandhi did not support the resolution on recruiting Indians for World War. 3. Consequent upon the breaking of the Salt Law by the Indian people, the Indian National Congress was declared illegal by the colonial rulers. Which of the statements given above are correct? (a) 1 and 2 only (b) 1 and 3 only (c) 2 and 3 only (d) 1, 2 and 3”,2019

A

67, (b), Statement 1 is correct: During the early 1900s, Mahatma Gandhi was instrumental in getting the indentured labour system abolished in the British Empire. He also supported Manilal Doctor’s campaign to get a resolution passed in the INC meeting condemning the indentured labour system in all British territories. Statement 2 is incorrect: Lord Chelmsford, the then Viceroy of India, invited Gandhi to Delhi at a War Conference. In order to gain the trust of the empire, Gandhi agreed to move people to enlist in the army for WW-I. Statement 3 is correct: Indian National Congress was declared illegal upon breaking of salt law. But this did not deter the satyagrahis who continued the movement.

68
Q

68,”The Gandhi-Irwin Pact included which of the following? (2020) 1. Invitation to Congress to participate in the Round Table Conference 2. Withdrawal of Ordinances promulgated in connection with the Civil Disobedience Movement 3. Acceptance of Gandhi’s suggestion for enquiry into police excesses. 4. Release of only those prisoners who were not charged with violence. Select the correct answer using the code given below: (a) 1 only (b) 1, 2 and 4 only (c) 3 only (d) 2, 3 and 4 only”,2020

A

68, (b), Statement 1 is correct: The Gandhi–Irwin Pact was a political agreement signed by Mahatma Gandhi and Lord Irwin, Viceroy of India, on 5 March 1931 before the Second Round Table Conference in London, where the Indian National Congress was invited to participate. Statement 2 is correct: Withdrawal of all ordinances issued by the Government of India imposing curbs on the activities of the Indian National Congress was agreed by the Viceroy. Statement 3 is incorrect: The demand for a public enquiry of excesses committed by police during the civil disobedience movement was rejected by the Viceroy. Statement 4 is correct: Release of all political prisoners, except those guilty of violence was also agreed.

69
Q

69,”With reference to 8th August 1942 in Indian history, which one of the following statements is correct? (2021) (a) The Quit India Resolution was adopted by the AICC. (b) The Viceroy’s Executive Council was expanded to include more Indians. (c) The Congress ministries resigned in seven provinces. (d) Cripps proposed an Indian Union with full Dominion Status once the Second World War was over.”,2021

A

69, (a), Option (a) is correct: Quit India Resolution was ratified in a Congress meeting at Gowalia Tank on 8th August 1942. Apart from QIM, other resolutions involved the Civil Disobedience Movement against British Rule. Mantra of “Do or Die” was adopted in this Session by Gandhiji. Option (b) is incorrect: More Indians were added to Viceroy’s executive council by the Government of India Act 1919. First time, an Indian was added in this council by the Government of India act 1909. Option (c) is incorrect: The Congress Ministries resigned from seven provinces in 1939 in protest because the then Viceroy Lord Linlithgow’s (1936-1944) action declared India to be belligerent in the WW-II without even consulting the Indian people. Option (d) is incorrect: Though Cripps proposed a dominion status to India and the failure of the cripps mission resulted in a nationwide Quit India Movement, this was not related to 8 August 1942.

70
Q

70,”In the context of Colonial India, Shah Nawaz Khan, Prem Kumar Sehgal and Gurbaksh Singh Dhillon are remembered as: (2021) (a) leaders of Swadeshi and Boycott Movement (b) members of the Interim Government in 1946 (c) members of the Drafting Committee in the Constituent Assembly (d) officers of the Indian National Army”,2021

A

70, (d), Indian National Army (INA) Trials/ Red Fort Trials: Between November 1945 and May 1946, the INA officers were brought before a court martial at the Red Fort in Delhi. At the Red Fort in Delhi, about ten court-martials were held in public. By holding open trials in the Red Fort, Claude Auchinleck, the commander-in-chief of the British-Indian army, hoped to sway public opinion against the INA. Major General Shah Nawaz Khan, Colonel Prem Kumar Sahgal and Colonel Gurbaksh Singh Dhillon were three of the senior-most officers of INA and trusted colleagues of Netaji. After considerable demonstrations and unrest in India, they were forced to be released after the British court-martialed them at Red Fort in 1945 and sentenced them to death. Congress leader and the country’s first Prime Minister, Jawaharlal Nehru, also got on board the INA officers’ legal defence team, along with party colleague Bhulabhai Desai and barrister Tej Bahadur Sapru.

71
Q

71,”Regarding Wood’s Dispatch, which of the following statements are true? (2018) 1. Grants-in-Aid system was introduced. 2. Establishment of universities was recommended. 3. English as a medium of instruction at all levels of education was recommended. Select the correct answer using the code given below: (a) 1 and 2 only (b) 2 and 3 only (c) 1 and 3 only (d) 1, 2 and 3”,2018

A

71, (a), Wood’s Dispatch (1854): Magna Carta of English Education in India. Government to take up the responsibility for the Education of masses. Hierarchy of Educational Institutions. Emphasise on Female, Vocational Education and teachers’ training. Secular Education in Government Institutions. Grants-in-aid to encourage private education. Recommended English as the Medium of Instruction in higher studies and vernaculars in school level. Universities with affiliations in three presidency towns of Calcutta, Bombay, and Madras. Statement 1 and 2 are correct: Grants-in-Aid system and Establishment of universities was very popular recommendations of Woods Dispatch, so option 1 and 2 are correct. However, if you look carefully at option 3, it talks about English as a medium of instruction at all levels of education. Dispatch was in 1854, at that time introducing English as a medium of instruction at all levels sounds unconvincing. By applying this crude logic, and taking the risk of eliminating option 3, we can get the answer.

72
Q

72,”Which of the following led to the introduction of English Education in India? (2018) 1. Charter Act of 1813 2. General Committee of Public Instruction, 1823 3. Orientalist and Anglicist Controversy. Select the correct answer using the code given below: (a) 1 and 2 only (b) 2 only (c) 1 and 3 only (d) 1, 2 and 3”,2018

A

72, (d), Statement 1 is correct: The Charter Act of 1813 permitted Christian missionaries to propagate English and preach their religion. Statement 2 is correct: As a president of the General Committee of Public Instruction, Lord Macaulay wrote a minute, where he Macaulay stressed the implementation of the English language as a medium of instruction through his minute. Statement 3 is correct: One faction of the Anglicists was in favour of English language as the medium, while the other faction was in favour of Indian languages (vernaculars). Charter Act of 1813: The idea of supporting educated Indians and promoting knowledge of contemporary sciences in the country was enshrined into the Charter Act of 1813. The Act required the Company to yearly approve one lakh rupees for this purpose. Even this meagre sum, though, was not made accessible until 1823, mostly due to the disagreement over the most appropriate strategy for this investment. The Charter Act of 1813 gave Christian missionaries the right to spread the English language and practise their religion. General Committee on Public Instruction (1823): A “General Committee of Public Instruction” was established by the Governor-General-in-Council in 1823 with the task of allocating one lakh rupees for educational purposes. The General Committee of Public Instruction also was not able to decide the medium of instruction by vote; due to lack of majority. Out of ten members of which Lord Macaulay was the president, five were supporters of English language or Anglicist as the medium of instruction and the rest were supporters of oriental or classic language or Classicists as a medium of instruction. This is the famous Anglicist and Classicist controversy. On February 2, 1835, Lord Macaulay, who served as head of the General Committee of Public Instruction, penned a minute in which he stated his position on the issue. Through his minute, Lord Macaulay emphasised the use of the English language as a medium of instruction. He believed that English was the ideal language for instruction. Orientalist-Anglicist controversy: Anglicists opinion: Exclusively suggested for spending on modern studies. Even the Anglicists were divided over the question of medium of instruction, where one faction was for English language as the medium, while the other faction was for Indian languages (vernaculars) for the purpose. Orientalists opinion: In order to prepare students for careers, Western sciences and literature should be taught, with a focus on advancing traditional Indian education.

73
Q

73,”With reference to educational institutions during colonial rule in India, consider the following pairs: Institution Founder: (2018) 1. Sanskrit College at Benaras - William Jones 2. Calcutta Madrasa - Warren Hastings 3. Fort William College - Arthur Wellesley Which of the pairs given above is/are correct? (a) 1 and 2 (b) 2 only (c) 1 and 3 (d) 3 only”,2018

A

73, (b), Statement 1 is incorrect: The Sanskrit College was established by Jonathan Duncan, the resident, at Benaras in 1791 for study of Hindu law and philosophy. Statement 2 is correct: The Calcutta Madrasah was established by Warren Hastings in 1781 for the study of Muslim law and related subjects. Statement 3 is incorrect: Fort William College was set up by Lord Richard Wellesley in 1800 for training of civil servants of the Company in languages and customs of Indians (closed in 1802).

74
Q

74,”Wellesley established the Fort William College at Calcutta because: (2020) (a) he was asked by the Board of Directors at London to do so (b) he wanted to revive interest in oriental learning in India (c) he wanted to provide William Carey and his associates with employment (d) he wanted to train British civilians for administrative purpose in India”,2020

A

74, (d), Fort William College (1800): Fort William College was founded on 10 July 1800 in Calcutta, British India. Fort William College was set up by Lord Richard Wellesley in 1800 It was set up for the training of civil servants of the Company in languages and customs of Indians. The court of directors of the British East India Company was not in support of a training college in Kolkata. Then it got closed in 1802 and a separate College was established in 1807 in England. NOTE: Question on Fort William College came in 2018. By simply reading solutions of that question, it was possible to attempt this question correctly, simply earning 2 marks !! This underscores the importance of solving and analysing PYQs.

75
Q

75,”Who among the following was associated as Secretary with Hindu Female School which later came to be known as Bethune Female School? (2021) (a) Annie Besant (b) Debendranath Tagore (c) Ishwar Chandra Vidyasagar (d) Sarojini Naidu”,2021

A

75, (c), Ishwar Chandra Vidyasagar (1820-1891) was appointed as the secretary of Bethune School/College established in 1849. Bethune School began as Hindu Female School in 1849 was renamed Bethune School in 1856. In 1856, the Government took charge of the Hindu Female School, later renamed Bethune School.

76
Q

76,”Which of the following parties were established by Dr. B.R. Ambedkar? (2012) 1. The Peasants and Workers Party of India. 2. All India Scheduled Castes Federation. 3. The Independent Labour Party. Select the correct answer using the codes given below: (a) 1 and 2 only (b) 2 and 3 only (c) 1 and 3 only (d) 1, 2 and 3”,2012

A

76, (b), Statement 1 is incorrect: The Peasants and Workers Party of India was founded in 1947. The party was founded in Maharashtra by Tulsidas Jadhav, Keshavrao Jedhe and others. Statement 2 and 3 are correct: Both The Independent Labour Party (1936) and All India Scheduled Castes Federation (1942) were established by Dr. B.R. Ambedkar.

77
Q

77,”Mahatma Gandhi undertook fast unto death in 1932, mainly because: (2012) (a) Round Table Conference failed to satisfy Indian political aspirations. (b) Congress and Muslim League had differences of opinion. (c) Ramsay Macdonald announced the Communal Award. (d) None of the statements (a), (b) and (c) given above is correct in this context.”,2012

A

77, (c), Communal Award (1932): The Prime Minister of Britain Ramsay MacDonald, announced the Communal Award in August 1932 to pursue the British policy of divide and rule. The Communal Award recognised the depressed classes as minorities and provided them separate electorates. Earlier the British had already given separate electorates to Muslims, Christians and Sikhs. Poona Pact 1932: B.R. Ambedkar negotiated the Poona Pact with Mahatma Gandhi in late September 1932. It was signed by Madan Mohan Malviya on behalf of Gandhi, and B. R. Ambedkar signed it on behalf of the depressed classes. The background to the Poona Pact was the Communal Award of August 1932, which, among other things, reserved 71 seats in the central legislature for the depressed classes. Gandhi, who opposed the Communal Award and believed it was a British effort to divide Hindus, started a fast unto death in order to have it overturned. He rejected the concept of a special electorate for the depressed classes. However, the number of seats set aside for them in provincial legislatures was increased to 147, and in the Central Legislature, they now make up 18% of all seats. The findings of the Indian Franchise Committee, also known as the Lothian Committee, served as the foundation for the communal award.

78
Q

78,”Annie Besant was: (2013) 1. Responsible for starting the Home Rule Movement 2. The founder of the Theosophical Society 3. Once the President of the Indian National Congress Select the correct statement/statements using the codes given below. (a) 1 only (b) 2 and 3 only (c) 1 and 3 only (d) 1, 2 and 3”,2013

A

78, (c), Annie Besant 1847-1933: She fought for the freedom of thought, secularism, women’s rights, birth control, workers’ rights and Fabian socialism. Following her encounter with Helena Blavatsky in 1889, Besant became a Theosophist. In 1893, she made her first trip to India with the Theosophical Society. She and Henry Steel Olcott co-founded the Theosophical Society - Adyar. She was the society’s president from 1907 to 1933. In Benares, Besant founded the Central Hindu College (CHC). She joined the Indian National Congress (INC) and demanded government action towards self-rule. She became the President of the Calcutta Session of INC, AD 1917. In 1916, Besant and Bal Gangadhar Tilak founded the All-India Home Rule League. She did not attend the AD 1920 Session at Nagpur due to growing differences with Gandhiji as she felt that the Government of India Act, 1919 was a means to free India. Newspapers: New India and Commonweal. She wrote the ‘Lotus Song’, an English version of the Gita. Statement 1 is correct: Annie Besant fought for the freedom of thought, secularism, women’s rights, birth control, workers’ rights and Fabian socialism. In 1916, Annie Besant and Tilak started the Home Rule Leagues at Madras and Belgaum respectively. Statement 2 is incorrect: The Theosophical Society was founded by Madame H. P. Blavatsky and Colonel Olcott in New York in 1875. In 1882, the headquarters of the Society were established in Adyar, near Madras (now Chennai) in India. Statement 3 is correct: Annie Besant became the President of the INC in Calcutta Session, 1917.

79
Q

79,”Consider the following statements: (2015) 1. The first woman President of the Indian National Congress was Sarojini Naidu. 2. The first Muslim President of the Indian National Congress was Badruddin Tyabji. Which of the statements given above is/are correct? (a) 1 only (b) 2 only (c) Both 1 and 2 (d) Neither 1 nor 2”,2015

A

79, (b), Statement 1 is incorrect: The first woman President of the Indian National Congress was Annie Besant in the 1917 Calcutta Session. Sarojini Naidu became the first Indian woman President of INC in 1925 Kanpur Session. Statement 2 is correct: Badruddin Tyabji became the first Muslim President of the Indian National Congress in the 1887 Madras Session.

80
Q

80,”Consider the following pairs: (2017) 1. Radhakanta Deb – first President of the British Indian Association 2. Gazulu Lakshminarasu Chetty – founder of the Madras Mahajan Sabha 3. Surendranath Banerjee – Founder of the Indian Association Which of the above pairs is/are correctly matched? (a) 1 only (b) 1 and 3 only (c) 2 and 3 only (d) 1, 2 and 3”,2017

A

80, (b), Pair 1 is correct: Raja Radhakanta Deb was the first President of the British Indian Association while Debendranath Tagore was its secretary. Pair 2 is incorrect: In 1884 Madras Mahajan Sabha was established by M. Veeraraghavachariar, G. Subramania Iyer and P. Anandacharlu. Pair 3 is correct: Surendranath Banerjee and Anand Mohan Bose founded the Indian Association of Calcutta in 1876.

81
Q

81,”He wrote biographies of Mazzini, Garibaldi, Shivaji and Shrikrishna; stayed in America for some time, and was also elected to the Central Assembly. He was (2018) (a) Aurobindo Ghosh (b) Bipin Chandra Pal (c) Lala Lajpat Rai (d) Motilal Nehru”,2018

A

81, (c), Lala Lajpat Rai, popularly known as Punjab Kesari, was a member of the Lal Bal Pal trio and served as the INC’s president at the 1920 Calcutta Special Session. He founded the Servants of the People Society in 1921, a non-profit welfare organization. He authored several notable works and led a non-violent march in protest to the Simon Commission, where he was brutally assaulted by the police and later succumbed to his injuries.

82
Q

82,”Which among the following events happened earliest? (2018) (a) Swami Dayanand established Arya Samaj. (b) Dinabandhu Mitra wrote Neeldarpan. (c) Bankim Chandra Chattopadhyay wrote Anandmath. (d) Satyendranath Tagore became the first Indian to succeed in the Indian Civil Services Examination.”,2018

A

82, (b), Neel Darpan is a Bengali play written by Dinabandhu Mitra in 1858–1859.

83
Q

83,”Consider the following pairs: (2019) Movement : Organization Leader 1. All India AntiUntouchability League : Mahatma Gandhi 2. All India Kisan Sabha : Swami Sahajanand Saraswati 3. Self-Respect Movement : E. V. Ramaswami Naicker Which of the pairs given above is/are correctly matched? (a) 1 only (b) 1 and 2 only (c) 2 and 3 only (d) 1, 2 and 3”,2019

A

83, (d), Pair 1: Mahatma Gandhi founded the All India Anti Untouchability League on 30 September, 1932. Pair 2: Sahajanand Saraswati was the first President of the All India Kisan Sabha. Pair 3: The self-Respect Movement was started by E.V. Ramaswamy Naicker in Tamil Nadu in 1925 with the aim of destroying the contemporary Hindu social order in its totality and creating a new, rational society without caste, religion, and god.

84
Q

84,”With reference to Indian National Movement, consider the following pairs: (2019) Person : Position held 1. Sir Tej Bahadur Sapru : President, All India Liberal Federation 2. K. C. Neogy : Member, The Constituent Assembly 3. P. C. Joshi : General Secretary, Communist Party of India Which of the pairs given above is/are correctly matched? (a) 1 only (b) 1 and 2 only (c) 3 only (d) 1, 2 and 3”,2019

A

84, (d), Pair 1: All India Liberal Federation was founded by Surendra Nath Banarjea and had leaders like Tej Bahadur Sapru, V. S. Srinivasa Sastri, and M. R. Jayakar, with Tej Bahadur Sapru as its president. Pair 2: K.C. Neogy was a member of the Constituent Assembly of India, a member of the first Cabinet of independent India, and the chairman of the first Finance Commission of India. Pair 3: P.C. Joshi was one of the early leaders of the communist movement in India and served as the first general secretary of the Communist Party of India from 1935–47.

85
Q

85,”With reference to the book “Desher Katha” written by Sakharam Ganesh Deuskar during the freedom struggle, consider the following statement: (2020) 1. It warned against the Colonial States hypnotic conquest of the mind. 2. It inspired the performance of swadeshi street plays and folk songs. 3. The use of ‘desh’ by Deuskar was in the specific context of the region of Bengal. Which of the statements given above are correct? (a) 1 and 2 only (b) 2 and 3 only (c) 1 and 3 only (d) 1, 2 and 3”,2020

A

85, (a), Both statements 1 and 2 are correct regarding the book “Desher Katha” and its impact during the Swadeshi Movement. Statement 3 is incorrect as Deuskar uses the word ‘desh’ in the context of the whole country.

86
Q

86,”The Vital-Vidhvansak, the first monthly journal to have the untouchable people as its target audience was published by: (2020) (a) Gopal Baba Walangkar (b) Jyotiba Phule (c) Mohandas Karamchand Gandhi (d) Bhimrao Ramji Ambedkar”,2020

A

86, (a), Gopal Baba Walangkar, also known as Gopal Krishna, was an early social activist working to alleviate the socio-economic oppression of the untouchable people in India. He published the first journal targeted at the untouchable people and wrote several articles and pamphlets advocating for their rights.

87
Q

87,”With reference to Madanapalle of Andhra Pradesh, which one of the following statements is correct? (2021) (a) Pingali Venkayya designed the tricolour Indian National Flag here. (b) Pattabhi Sitaramaiah led the Quit India Movement of Andhra region from here. (c) Rabindranath Tagore translated the National Anthem from Bengali to English here. (d) Madame Blavatsky and Colonel Olcott set up headquarters of Theosophical Society first here.”,2021

A

87, (c), Rabindranath Tagore translated the National Anthem from Bengali to English at Jorasanko Thakur Bari, the ancestral home of the Tagore family in Kolkata.

88
Q

88,”Who among the following is associated with ‘Song from Prison’, a translation of ancient Indian religious lyrics in English? (2021) (a) Bal Gangadhar Tilak (b) Jawaharlal Nehru (c) Mohandas Karamchand Gandhi (d) Sarojini Naidu”,2021

A

88, (c), Mahatma Gandhi proclaimed the slogan ‘do or die’ at the historic August meeting at Gowalia Tank in Bombay during the Quit India Movement. He was arrested on August 9, 1942, and later undertook a 21-day fast in protest against the government’s actions.

89
Q

89,”With reference to Indian History, the Members of the Constituent Assembly from the Provinces were: (2013) (a) directly elected by the people of those Provinces (b) nominated by the Indian National Congress and the Muslim League (c) elected by the Provincial Legislative Assemblies (d) selected by the Government for their expertise in constitutional matters”,2013

A

89, (c), The idea of a constituent assembly was first put forward by MN Roy. The Indian National Congress first formally requested a constituent assembly in 1935. The Constituent Assembly was constituted in November 1946 under the Cabinet Mission Plan. It had a total strength of 389 members, with seats allotted based on population proportion.